返回列表 发帖

. 请教LSAT中一种逻辑的问法

Which one of the following, if true, most calls into question the XXX’argument?fficeffice" />

我理解成削弱,结果答案总是做错,搞不清楚到底是加强还是削弱XXX的观点。

For example:

The gray squirrel, introduced into local woodlands ten years ago, threatens the indigenous population of an endangered owl species, because the squirrels’ habitual stripping of tree bark destroys the trees in which the owls nest. Some local officials have advocated setting out poison for the gray squirrels. The officials argue that this measure, while eliminating the squirrels, would pose no threat to the owl population, since the poison would be placed in containers accessible only to squirrels and other rodents.

Which one of the following, if true, most calls into question the officials’ argument?

(A) One of the species whose members are likely to eat the poison is the red squirrel, a species on which owls do not prey.

(B) The owls whose nesting sites are currently being destroyed by the gray squirrels feed primarily on rodents.

(C) No indigenous population of any other bind species apart from the endangered owls is threatened by the gray squirrels.

(D) The owls that tare threatened build their nests in the tops of trees, but the gray squirrels strip away back from the trunks. (E) The officials’ plan entails adding the poison to food sources that are usually eaten by rodents but not by other animals.

收藏 分享

削弱fficeffice" />

TOP

返回列表

站长推荐 关闭


美国top10 MBA VIP申请服务

自2003年开始提供 MBA 申请服务以来,保持着90% 以上的成功率,其中Top10 MBA服务成功率更是高达95%


查看